Find all triples $ (x,y,z)$ of real numbers that satisfy the system of equations \[ \begin{cases}x^3 = 3x-12y+50, \\ y^3 = 12y+3z-2, \\ z^3 = 27z + 27x. \end{cases}\] Razvan Gelca.
Problem
Source: USA TST 2009 #7
Tags: algebra, polynomial, trigonometry, system of equations, algebra proposed, Hi
20.07.2009 13:05
This is my solution: we have 2 cases: case1:$ x\ge 2$,from 1 we have $ y\le 4$,from 3 we have $ z\ge 6$ so from 2 we have $ y\ge 4$ so x=2,y=4,z=6 case2:$ x\le 2$,from 1 we have $ y\ge 4$,from 3 we have $ z\le 6$ so from 2 we have $ y\le 4$ so x=2,y=4,z=6 Q.e.d
20.07.2009 18:42
tuandokim wrote: This is my solution: we have 2 cases: case1:$ x\ge 2$,from 1 we have $ y\le 4$,from 3 we have $ z\ge 6$ so from 2 we have $ y\ge 4$ so x=2,y=4,z=6 case2:$ x\le 2$,from 1 we have $ y\ge 4$,from 3 we have $ z\le 6$ so from 2 we have $ y\le 4$ so x=2,y=4,z=6 Q.e.d nice solution
20.07.2009 19:50
Another solution could be just setting x=a+2, y=b+4, z=c+6. If one crunches the numbers, and observes the trivial inequality, he/she will find a=b=c=0, so x=2, y=4, z=6.
04.08.2009 18:57
set $ z=3u$ and take $ x$ from the last equation into the first one, so that it turns into $ (u^3-3u)^3=3(u^3-3u)-12y+50$ and $ y^3=12y+9u-2$... take $ y=w+4$, and this system turns to $ (u^3-3u)^3-3(u^3-3u)-2=-12w$ and $ w^3+12w^2+36w=9u-18$... this can be rewritten as $ (u-2)(u^4+u^3-3u^2-2u+1)^2=-12w$ and $ w(w+6)^2=9(u-2)$... from this it's easy to conclude that $ u=2, w=0$, or equivalently, $ z=6, y=4, x=2$
09.10.2009 05:19
Great solutions, but how did you guys get that 2, 4, 6 worked? was it just guess? Another solution involves using the cos(3*theta) formula to kill the cubic, which makes it immediately evident why 2 4 and 6 are solutions.
24.12.2009 22:28
Rewrite the equations as the following: $ 12(4 - y) = (x - 2)(x + 1)^{2}$ $ (1)$, $ 3(z - 6) = (y - 4)(y + 2)^{2}$ $ (2)$, $ 27(x - 2) = (z - 6)(z + 3)^{2}$ $ (3)$. $ (2)$, $ (3)$ gives $ (y - 4)(x - 2)\geq{0}$ (Or $ y = - 2$, which leads to a contradiction), but from $ (1)$ we get: $ (4 - y)(x - 2)\geq{0}$. (Or $ x = - 1$, which also leads to a contradiction). So $ 0\leq{4 - y)(x - 2)\leq{0}}$. It means $ y = 4$ or $ x = 2$, but both of them lead to an unique solution: $ (x,y,z) = (2,4,6)$.
09.02.2012 20:07
ZerAn wrote: Great solutions, but how did you guys get that 2, 4, 6 worked? was it just guess? I have the same question.
28.03.2013 17:55
Here's a slightly more deterministic solution. Substitute $y=2r, z=3s$ to make more of the coefficients match. Then we get $x^3-3x=50-24r$ $r^3-3r=(9s-2)/8$, $s^3-3s=x$. The solution $x=r=s=2$ pretty much jumps out. We note that $x^2-3x-2=(x-2)(x+1)^2$, so $(x-2)(x+1)^2=24(2-r)$, $(r-2)(r+1)^2=9(s-2)/8$, $(s-2)(s+1)^2=(x-2)$. If any of $x,r,s$ equals 2, then they all do. If not, multiplication yields that $(x+1)^2(r+1)^2(s+1)^2=-27$, a contradiction.
30.03.2013 17:25
Let $x=2a$, $y=4b$ and $z=6c$ (this is motivated by observing $a=b=c=1$). Then, the givens rearrange to \begin{align*} a(4a^2-3) &= 25-24b \\ b(4b^2-3) &= \frac{9c-1}{8} \\ c(4c^2-3) &= a \end{align*} Note $c < 1 \implies a \le 1 \implies 25-24b \le 1 \implies b \le 1 \implies \frac{9c-1}{8} \le 1 \implies c \ge 1$ and $c > 1 \implies a > 1 \implies 25-24b > 1 \implies b < 1 \implies \frac{9c-1}{8} < 1 \implies c < 1$. So $a=b=c=1$ is the only solution; i.e. $(x,y,z) = (2,4,6)$. ========================================================== This bounding approach is pretty motivated by the polynomial $f(x) = 4x^3-3x$, which behaves very nicely with respect to bounds; in fact, I was trying to get a solution using the identity $\cos 3x = 4\cos^3 x - 3 \cos x$ which is why I attempted the bounding in the first place. ZerAn wrote: Great solutions, but how did you guys get that 2, 4, 6 worked? was it just guess? Many substitutions later. Not realizing that this was essentially bounding, I tried a lot of different substitutiotns and one of which eventually led to solving $u-v = \tfrac{u^3-50}{3}$, $v-w = \tfrac{v^3+128}{192}$, $w-u=\tfrac{w^3}{27}$. At this point I decided to start looking for integer solutions and finally noticed $(u,v,w) = (2,16,-6)$ which gave $(x,y,z) = (2,4,6)$. In retrospect I think I should have tried the guessing first. So I guess the main difficulty of the problem was finding the triple in the first place, and afterwards it was just a matter of careful manipulations.
12.05.2016 07:31
07.03.2017 03:41
Answer: The only triple which satisfies our condition is $(x, y, z)=(2, 4, 6)$. (Proof) Suppose $(x-2)(y-4)(z-6) \ne 0,$ as otherwise we get $(x, y, z)=(2, 4, 6)$ as claimed. Note that the given equations may be re-written as: \[ \begin{cases} -12(y-4)=(x-2)(x-1)^2, \\ 3(z-6)=(y-4)(y+2)^2, \\ 27(x-2)=(z-6)(z+3)^2. \end{cases}\]Taking the product and cancelling the common (non-zero) factor $(x-2)(y-4)(z-6),$ we see that both sides have an opposite sign unless $(x-1)(y+2)(z+3)=0$. Evidently, the latter cannot occur as $$x=1 \Longrightarrow y=4 \Longrightarrow z=6 \Longrightarrow x=2,$$a contradiction! Similar arguments apply for the other two factors.
29.08.2019 02:46
We claim the only solution is $(x,y,z)=\boxed{(2,4,6)}$, which can easily be checked to work. To show that this is the only solution, we have the following key lemma. Lemma: We have \begin{align*} x\ge 2 &\iff x^3-3x\ge 2 \\ y\ge 4 &\iff y^3-12y\ge 16 \\ z\ge 6 &\iff z^3-27z\ge 54, \end{align*}and similar statements with all the inequality directions reversed. Proof: The proof is ``you just check it''. $\blacksquare$ The solution to the problem is now easy. We see that \begin{align*} x\ge 2 &\iff z^3-27z\ge 54 \\ &\iff z\ge 6 \\ &\iff y^3-12y\ge 16 \\ &\iff y\ge 4 \\ &\iff x^3-3x\le 2 \\ &\iff x\le 2, \end{align*}so $x=2$. This easily gives that $(x,y,z)=(2,4,6)$, as desired.
22.10.2019 22:12
We have \begin{align*} x\ge 2 &\implies \frac{z^3-27z}{27} \ge 2 \implies z^3-27z-54\ge 0 \implies z\ge 6 \text{ or } z=-3. \\ z\ge 6 &\implies y^3-12y = 3z-2 \ge 16 \implies y^3-12y-16 \ge 0 \implies y\ge 4 \text{ or } y=-2. \\ y\ge 4 &\implies x^3-3x-50=-12y \le -48 \implies x^3-3x-2\le 0 \implies x\le 2 \text{ or } x=-1. \end{align*}It is easy to check that none of $z=-3$ or $y=-2$ or $x=-1$ produce valid solutions. Therefore, $x\ge 2 \implies x\le 2$. We can make a similar inequality chain to show that $x\le 2\implies x\ge 2$. Therefore, $x=2$, which produces the solution $(x,y,z)=(2,4,6)$.
30.10.2019 20:37
Substituting $x=a+2$, $y=b+4$, $z=c+6$, we get the equations $a(a+3)^2=-12b$, $b(b+6)^2=3c$, $c(c+9)^2=27a$. So, $a,b$ are of opposite sign, $b,c$ are of the same sign, and $c,a$ are of the same sign. The only way this can happen is when $a=b=c=0$, so our only solution is $(2,4,6)$.
01.02.2020 19:41
17.03.2020 06:13
hmm my solution seems to be identical to william122's but whatever I claim that $(x, y, z)=(2, 4, 6)$ are the only solutions. It is trivial to verify that these work. To prove that they are the only ones, let $x-2=a$, $y-4=b$, $z-6=c$. The equations rearrange to $a(a+3)^2=-12b, b(b+6)^2=3c$, and $c(c+9)^2=27a$. If one of $(a, b, c)$ is $0$ then clearly they are all $0$. If they are all nonzero, then note $\frac{a}{b}<0$, $\frac{b}{c}>0$, and $\frac{c}{a}>0$, but by looking at signs, this is clearly impossible. Hence, $(a, b, c)=(0, 0, 0)$, so we are done. $\blacksquare$
05.05.2020 00:42
The answer is $(x, y, z) = (2, 4, 6)$, which we will show is the only valid solution. Set $x = a + 2$, $y = b+4$, and $z = c + 6$. Expanding the given equations gives $a(a+3)^2 = -12b, b(b+6)^2 = 3c$, and $c(c+9)^2 = 27a$. If $a = 0$, then the solution set follows. Else, if $a > 0$, then we must have $b < 0$, meaning $c < 0$, which in turn implies $a < 0$, contradiction. Otherwise, if $a < 0$, then $b > 0$, which means $c > 0$, so $a > 0$, another contradiction. $\blacksquare$
05.05.2020 00:45
doing equality i see
19.09.2020 02:45
The answer is $\boxed{(x,y,z) = (2,4,6).}$ Now let $a = x-2, b = y-4, z - x-6$. Substituting, the problem tells us that $a(a+3)^2 = -12b, b(b+6)^2 = 3c,$ and $c(c+9)^2 = 27a$. Note that clearly either one of $(a,b,c) =0$ forces all three to be $0$, from which we derive our equality set. Otherwise, note that $a$ and $b$ have opposite signs, $b$ and $c$ have the same sign, and $a$ and $c$ have the same sign, so $a$ and $b$ must have the same sign, a contradiction. Remarks: Hopefully not all equality problems end up like this
08.08.2023 17:52
Great problem! Notice the equality case (2,4,6), hence we are motivated to let $x=a+2,y=b+4,z=c+6$ and try to show they're all 0. We get $$a^3+6a^2+12a+8=(3a+6)-(12b+48)+50\iff a^3+6a^2+9a=-12b\iff a(a+3)^2=-12b,$$$$b^3+12b^2+48b+64=(12b+48)+(3c+18)-2\iff b^3+12b^2+36b=3c\iff b(b+6)^2=3c,$$$$c^3+18c^2+108c+216=(27c+162)+(27a+54)\iff c^3+18c^2+81c=27a\iff c(c+9)^2=27a.$$Multiplying them together, the LHS has sign abc while the RHS has opposite sign -abc; hence abc=0. Now, caseworking on any one of them being 0, we get that each of a,b,c=0, as desired. $\blacksquare$
22.08.2023 20:32
The only triplet is $(x, y, z) = (2, 4, 6)$, which works. To prove this is the only triplet, let $x = a + 2, y = b + 2, z = c + 2$. Then our equations rewrite as $$\begin{aligned} a(a + 3)^2 &= -12b, \\ b(b + 6)^2 &= 3c, \\ c(c + 9)^2 &= 27a. \end{aligned}$$If any of $a, b, c$ equals $0$, it's easy to see they must all equal $0$ (which works). Else, If $a > 0$, then $b < 0$ from the first equation, and $c < 0$ from the second equation, which violates the third equation If $a < 0$, then $b > 0$ from the first equation, and $c > 0$ from the second equation, which violates the third equation Thus, $(a, b, c) = (0, 0, 0) \implies (x, y, z) = (2, 4, 6)$, as desired.
15.11.2023 02:43
22.12.2023 23:57
We claim that the only triple is $(x,y,z) = \boxed{(2,4,6)}$, which clearly works. Substitute in $(x,y,z)=(a+2, b+4, c+6)$ to get \begin{align*} a(a+3)^2 &= -12b,\\ b(b+6)^2 &= 3c,\\ c(c+9)^2 &= 27a. \end{align*} If $a=0$, then we have $b=c=0$, which produces our claimed solution. If $a>0$, we have $b<0 \implies c<0 \implies a<0$, contradiction. If $a<0$, we have $b>0 \implies c>0 \implies a>0$, contradiction. Thus, there are no other solutions than the one claimed.
24.12.2023 14:38
USA TST 2009/7 wrote: wrote: Find all triples $ (x,y,z)$ of real numbers that satisfy the system of equations \[ \begin{cases}x^3 = 3x-12y+50, \\ y^3 = 12y+3z-2, \\ z^3 = 27z + 27x. \end{cases}\] Fun(ny?) Problem realised never typed this up re-write the equations as follows $(x-2)(x+1)^2=-12(y-4)$ $(y-4)(y+2)^2=3(z-6)$ $(z-6)(z+3)^2=27(x-2)$ if $x \neq 2, y \neq 4, z \neq 6$ then multiply all of the equations, we get one side positive while other negative, contradiction!. Hence forcing $x=2,y=4,z=6$
26.12.2023 22:00
We claim that the only solution is $\boxed{(x,y,z)=(2,4,6)}$. Let $x=a,y=2b,z=3c$. Substituting them, we get the following equations: \[ \begin{cases}a^3-3a=50-4b, \\ b^3-3b=\frac{9c-2}8, \\ c^3-3c=a. \end{cases}\]Note that the function $f(x)=x^3-3x$ is a cubic polynomial function. $f'(x)=0$ has two solutions, namely $x=\pm1$. $f(1)=-2,f(-1)=f(2)=2$. If $x<-1$ then $f(x)<f(-1)=f(2)=2$ and if $-1<x<1$, then $2=f(-1)>f(x)>f(1)=-2$. If $1<x<2$ then $f(x)<f(2)$. This means that if $x<2$ then $f(x)\leq f(2)$ and if $x>2$ then $f(x)>f(2)$. So, if $a>2$, then $50-24b>2$, giving $b<2$. If $b<2$, then $\frac{9c-2}8\leq2$, which means $c\leq2$. If $c\leq2$ then $a\leq2$, contradiction. If $a<2$, then $50-24b<2$, giving $b>2$. If $b>2$, then $\frac{9c-2}8>2$, which means $c>2$. If $c>2$ then $a>2$, contradiction. This means $a=2$ is forced, giving $b=2$ and $c=2$, which clearly work. So, $(x,y,z)=(2,4,6)$ is the only solution.
06.01.2024 00:29
Answer is $(2, 4, 6)$. Substitute in \[(x, y, z) = (\alpha + 2, \beta + 4, \gamma + 6)\]\[(\alpha + 2)^3 = 3\alpha - 12\beta + 8\]\[(\beta + 4)^3 = 12\beta + 3\gamma + 64\]\[(\gamma + 6)^3 = 27\alpha + 27\gamma + 216\]\[\implies\]\[\alpha(\alpha +3)^2 = -12\beta\]\[\beta(\beta + 6)^2 = 3\gamma\]\[\gamma(\gamma + 9)^2 = 27\alpha\]\[\implies\]\[\alpha\beta\gamma(\alpha +3)^2(\beta + 6)^2(\gamma + 9)^2 = -972\alpha\beta\gamma\]FTSOC, assume that $\alpha\beta\gamma \neq 0$. Then dividing by $\alpha\beta\gamma$ results in \[(\alpha +3)^2(\beta + 6)^2(\gamma + 9)^2\ = -972\]which is a contradiction due to the trivial inequality. Hence, $\alpha\beta\gamma = 0$. Plugging $0$ into $\alpha$, $\beta$, or $\gamma$, we can clearly see that it makes the rest of the variables $0$, so our answer for $(x, y, z)$ is $(2, 4, 6)$.
21.01.2024 06:25
We claim our only solution is $\boxed{(x,y,z)=(2,4,6)}$. Motivated by this answer, we notice our equations can be rearranged to be \begin{align*} (x-2) \cdot ((x-2)+5)^2 = -12(y-4) \\ (y-4) \cdot ((y-4)+10)^2 = 3(z-6) \\ (z-6) \cdot ((z-6)+15)^2 = 27(x-2). \end{align*} Multiplying these, we see that we must have one of $x=2$, $y=4$, or $z=6$ to avoid a trivial inequality contradiction. However, any one of the these imply the other two, which finishes. $\blacksquare$
12.02.2024 10:15
We claim that $(x, y, z) = (2, 4, 6)$ which clearly works. Note the system of equations rearranges to \begin{align*} (x + 1)^2 (x - 2) &= -12(y - 4) \\ (y + 2)^2 (y - 4) &= 3(z - 6) \\ (z + 3)^2 (z - 6) &= 27(x - 2). \end{align*}Multiplying yields \[(x + 1)^2 (y + 2)^2 (z + 3)^2 (x - 2)(y - 4)(z - 6) = -972(x - 2)(y - 4)(z - 6).\]Now if $x = 2$, $y = 4$ or $z = 6$ then it is easily seen that $(x, y, z) = (2, 4, 6)$ by direct substitution. Otherwise \[(x + 1)^2 (y + 2)^2 (z + 3)^2 = -972,\]a contradiction.
15.02.2024 04:45
The only solution is $\boxed{(2, 4, 6)}$. Consider rearranging as, \begin{align*} (x-2)(x+1)^2 &= 12(4 - y)\\ (y-4)(y+2)^2 &= 3(z - 6)\\ (z-6)(z+3)^2 &= 27(x - 2) \end{align*}Multiplying we have, \begin{align*} (x-2)(y-4)(z-6)(x+1)^2(y+2)^2(z+3)^2 &= -927(x-2)(y-4)(z-6) \end{align*}Clearly if all of the factors $(x-2)$, $(y-4)$ and $(z-6)$ are nonzero we have no solutions. However if we have $x = 2$, $y = 4$ or $z = 6$ we find the claimed solution so we are done.
16.03.2024 08:44
The hardest part of this problem is finding the solution itself. We claim that the only solution to this system of equations is $(x,y,z) = (2,4,6).$ This can clearly be shown to work just by plugging in. Now we show that this is the only solution. Inspired by the equality case, we set $x = p + 2, y = q + 4,$ and $z = r+6.$ After plugging in and reducing the equation, we get the following systems of equations: \begin{align*} p(p+3)^2 &= -12q \\ q(q+6)^2 &= 3r \\ r(r+9)^2 &= 27p. \end{align*}If any of $p,q,r$ are equal to $0,$ then all of them must be $0$ simply by plugging in, giving us the solution claimed at the beginning of our solution. Henceforth assume that $p,q,r$ are nonnegative. The first equation rearranges to \[ -\frac{p}{12q} = \left(\frac{1}{p+3}\right)^2, \]so \[ -\frac{81p}{q} = 972 \cdot \left(\frac{1}{p+3}\right)^2. \]Meanwhile, the second and third equations multiply to \[ qr((q+6)(r+9))^2 = 81pr, \]which, since $r \ne 0,$ rearranges to \[ \frac{81p}{q} = ((q+6)(r+9))^2. \]Adding our two equations together, we get \[ 972 \cdot \left(\frac{1}{p+3}\right)^2 + ((q+6)(r+9))^2 = 0. \]Since the square of a real number is nonnegative, all of the squares in the above expression are $0.$ In particular, $\frac{1}{p+3} = 0,$ which is impossible. Therefore, $p = q = r = 0,$ and so the only solution to our system of equations is $\boxed{(x,y,z) = (2,4,6)},$ as claimed.
23.07.2024 06:42
Answer: $(2,4,6).$ We can check this works. Substitute $x=X+2,y=Y+4,z=Z+6.$ The equations become \begin{align*}&X(X+3)^2=-12Y,\\&Y(Y+6)^2=3Z,\\&Z(Z+9)^2=27X.\end{align*}Now it is easy to see that if one of $X,Y,Z$ is zero then all are. But multiplying all together gives $-972XYZ=XYZ(X+3)^2(Y+6)^2(Z+9)^2,$ impossible if $XYZ\ne 0.$ Thus $(X,Y,Z)=(0,0,0)$ so $(x,y,z)=(2,4,6).$
06.08.2024 04:21
The solution $(x, y, z) = (2, 4, 6)$ is the only one. Now note that the equations are equivalent to: $$(x + 1)^2 (x - 2) = -12(y - 4)$$$$(y + 2)^2 (y - 4) = 3(z - 6)$$$$(z + 3)^2 (z - 6) = 27(x - 2)$$It is easy to see that $x = 2$, $y = 4$, or $z = 6$ are independently sufficient enough to yield $(2, 4, 6)$ as a solution, so assume $x \ne 2, y \ne 4, z \ne 6$. Then multiplying the three equations and dividing by $(x - 2)(y - 4)(z - 6)$ gives $$(x + 1)^2 (y + 2)^2 (z + 3)^2 = -12 \cdot 3 \cdot 27$$which is clearly impossible. So $(x, y, z) = (2, 4, 6)$ is the only solution and we are finished.
03.09.2024 17:07
We claim the only solution is $(x, y, z) = (2, 4, 6)$, which clearly works. Let $x = a+2, y = b+4, z = c+6$. Then the equations simplify as $$a(a+1)^2 = -12b$$$$b(b+2)^2 = 3c$$$$c(c+3)^2 = 27a.$$Now assume none of $a, b, c$ are zero, since if one is then clearly all are. Then we get $$(a+1)^2 = -\frac{12b}{a}, (b+2)^2 = \frac{3c}b, (c+3)^2 = \frac{27a}c.$$In particular, if $sgn(k)$ denotes the sign of $k$, $$sgn(a) \ne sgn(b), sgn(b) = sgn(c), sgn(c) = sgn(a),$$a clear contradiction. Therefore the only working solution is $(x, y, z) = (a+2, b+4, c+6) = (2, 4, 6)$. $\square$
07.09.2024 17:08
Answer: $(x,y,z)=(2,4,6)$ Solution: Let $x=a+2$ , $y=b+4$ , $z=c+6$ Our system of equations transforms into the following: \[ \begin{cases}a^3+6a^2+9a=-12b ...(1) , \\ b^3+12b^2+36b=3c ...(2) , \\ c^3+18c^2+81c=27a ...(3) \end{cases}\] Since we want to show that $a=b=c=0$ we will prove it by a contradiction. AFTSOC $a \neq 0$ $\textbf{Case 1.}$ $a >0$ $...(*)$ From $(1) \implies -12b \stackrel{(1)}{=}a^3+6a^2+9a >0+0+0=0 \implies -12b>0 \implies -b>0 \implies b<0$ $...(**)$ Combining $(2)$ and $(**) \implies 0=0+0+0 \stackrel{(**)}{<} b^3+12b^2+36b \stackrel{(2)}{=} 3c \implies 0<3c \implies 0<c$ $...(***)$ Combining $(3) , (*) $ and $(***)$ we get: $0=0+0+0 \stackrel{(***)}{<} c^3+18c^2+81c \stackrel{(3)}{=} 27a \stackrel{(*)}{<} 27 \cdot 0 =0 \implies 0<0 \rightarrow \leftarrow $. $\textbf{Case 2.}$ $a<0$ $...(@)$ From $(1) \implies -12b \stackrel{(1)}{=}a^3+6a^2+9a < 0+0+0=0 \implies -12b<0 \implies -b<0 \implies b>0$ $...(@@)$ Combining $(2)$ and $(@@) \implies 0=0+0+0 \stackrel{(@@)}{>} b^3+12b^2+36b \stackrel{(2)}{=} 3c \implies 0>3c \implies 0>c$ $...(@@@)$ Combining $(3) , (@)$ and $(@@@)$ we get: $0=0+0+0 \stackrel{(@@@)}{>} c^3+18c^2+81c \stackrel{(3)}{=} 27a \stackrel{(@)}{>} 27 \cdot 0 =0 \implies 0>0 \rightarrow \leftarrow $. Hence $\boxed{a=0}$ from $(1)$ we find that $\boxed{b=0}$ and finally from $(2)$ we find that $\boxed{c=0}$. $\blacksquare$